Answer: B

This is a weaken question. The experts' proposal involves expanding opportunities for women so that affected women will have fewer children, thus limiting the size of the population peak that the same experts have predicted. We're looking for a choice that suggests that the proposal will not limit the size of the peak or cause it to occur earlier. Consider each one:

(A) This is irrelevant. If the proposal 20 years ago was not followed (and we don't know whether it was or not) the fact that it was made doesn't tell us whether it would be effective or not today.
(B) This is correct. The proposal is based on the assumption that greater opportunities could be arranged for women in developing countries. If the maximum possible opportunities are already present, and have been built into the prediction, the proposal will not bring about any changes in the population peak.
(C) The argument is concerned primarily with developing countries, not industrialized countries.
(D) It doesn't matter whether the goal is desirable; the question is concerned with whether the goal will be attained.
(E) This is outside of the scope. It may be true, but it doesn't bear on whether the proposal will have the desired effect.